This is posted in classical physics, however.. and in any case if it is undergoing simple harmonic motion then it isn't a quantum harmonic oscillator, so I don't see any reason to be messing around with Schrodingers.
Hi, I think for your second question you are right. It blows up, but its an artifact of assuming that the wire is infinitely tiny.
See for interesting discussion:
http://physics.stackexchange.com/questions/16520/potential-energy-of-a-charged-ring...
Hi, unfortunately you can't add the angular velocities of the wheels like that. Just like your car has four wheels going 60 mph each, but the car doesn't go 240 mph, in your launcher, the second wheel just allows there to be good grip and provide more power with a second motor.
You are...
I believe it should affect (increase) the field.. If you imagine some fixed number of field lines, the tapered ends would concentrate the field lines, increasing flux, which should increase magnetic field strength. This effect is used the in design of electromagnets, where the tip of the iron...
y(x,t)=Asin(ωt+kx) is the equation of motion for a simple harmonic oscillator.
You get this by solving Newton's force law..
F=ma=-kx \\
ma+kx=0 \\
a+\frac{k}{m}x=0
Or you can write
\ddot{x}+\frac{k}{m}x=0
This is a differential equation, solved by Asin(ωt+kx), where \omega...
You could think of the secant as the average slope, which you get from rise/run=4/1=4, which you did correctly.
With lim h->0 (f(x+h)-f(x)) / h, you are calculating the slope of the tangent line at f(x). So, when you calculated y', you found found the tangent at point P.
For b, you...